OSPE
OSPE
He is
compliant, has a stable INR, and denies bleeding or bruising. He is diagnosed with a urinary tract infection
and is prescribed sulfamethoxazole/trimethoprim. What are the expected effects on his warfarin therapy?
Correct answer = B.
Sulfamethoxazole/trimethoprim can increase the concentration of warfarin, leading to
Enhanced anticoagulant effects and an elevated risk of bleeding.
2. A patient develops urinary retention after an abdominal surgery. Urinary obstruction was ruled out in this
patient. Which strategy would be helpful in promoting urination?
Correct answer = C
Inhibiting cholinesterase enzyme increases acetylcholine levels, facilitating bladder emptying And
helping alleviate urinary retention after surgery.
3. Which is correct regarding activation of receptors on the effector organs in the ANS?
Correct answer = A.
Acetylcholine activates muscarinic receptors on effector organs in the autonomic nervous System,
influencing parasympathetic responses.
4. A pharmacokinetic study of a new antihypertensive drug is being conducted in healthy human volunteers.
The half-life of the drug after administration by continuous intravenous infusion is 12 hours. Which of the
following best approximates the time for the drug to reach steady state?
A. 24 hours
B. 48 hours
C. 72 hours
D. 120 hours
E. 240 hours
Correct answer = B.
It takes approximately 4-5 half-lives for a drug to reach steady state. With a half-life of 12 hours, the time for the drug
to reach steady state is around 48 hours (0ption B).
5. A new antihypertensive drug was tested in an animal model of hypertension. The drug when given alone reduces
blood pressure in the animal. Norepinephrine when given in the presence of this drug did not cause any significant
change in blood pressure or heart rate in the animal. The mechanism of action of the new drug is similar to which
of the following agents?
A. Doxazosin
B. Clonidine
C. Atenolol
D. Carvedilol
Correct answer = D.
Carvedilol, like the new drug, is a non-selective beta-blocker with alpha-blocking properties. It can block the effects of
norepinephrine, leading to reduced blood pressure
6. A 47-year-old woman presents to the emergency room with severe bleeding. Upon evaluation of the
medical record, you discover that she takes dabigatran for a history of multiple DVTs. What is the appropriate
reversal agent to administer to the patient at this time?
A. Protamine
B. Vitamin K
C. Idarucizumab
D. A reversal agent does not exist for this medication
Correct answer = C.
Dabigatran is a direct thrombin inhibitor, and its reversal agent is idarucizumab (0ption C),
which rapidly and specifically reverses the anticoagulant effects of dabigatran in emergency situations.
7. A 58-year-old man receives intravenous alteplase treatment for acute stroke. Five minutes following
completion of alteplase infusion, he develops angioedema. Which of the following drugs may have increased
the risk of developing angioedema in this patient?
A. ACE inhibitor
B. GP IIb/IIIa receptor antagonist
C. Phosphodiesterase inhibitor
D. Thiazide diuretic
Correct answer = A.
ACE inhibitors (Option A) are known to increase the risk of angioedema, and combining them
With alteplase, a thrombolytic agent, can further elevate this risk.
8. Which of the following drugs decreases cholesterol synthesis by inhibiting the enzyme 3hydroxy-3-
methylglutaryl coenzyme A reductase?
A. Fenofibrate
B. Cholestyramine
C. Lovastatin
D. Gemfibrozil
Correct answer = C.
Lovastatin (Option C) is an HMG-CoA reductase inhibitor, commonly known as a statin, which
decreases cholesterol synthesis by inhibiting the enzyme HMG-CoA reductase.
9. A 40-year-old female is undergoing infertility treatments. Which drug might be included in her treatment
regimen?
A. Cabergoline
B. Follitropin
C. Methimazole
D. Vasopressin
Correct answer = B.
Follitropin (Option B) is a fertility medication that stimulates the ovaries to produce eggs. It is
used in infertility treatments to promote ovulation.
10. A 70-year-old woman is being treated with raloxifene for osteoporosis. Which is a concern with this therapy?
A. Breast cancer
B. Endometrial cancer
C. Venous thrombosis
D. Hypercholesterolemia
Correct answer = C.
Venous thrombosis (Option C) is a concern with raloxifene therapy. This selective estrogen
receptor modulator is used for osteoporosis but may increase the risk of blood clots.
A 23-year-old man presents with acute appendicitis that ruptures shortly after admission. He is taken to the
operating room for surgery, and postsurgical cultures reveal Escherichia coli and Bacteroides fragilis,
susceptibilities pending. Which of the following provides adequate empiric coverage of these two pathogens?
A. Cefepime
B. Piperacillin/tazobactam
C. Aztreonam
D. Ceftaroline
Correct answer = B.
Piperacillin/tazobactam (Option B) provides broad-spectrum coverage against both Gram-negative (Escherichia coli)
and anaerobic (Bacteroides fragilis) organisms
2. A 9-year-old girl has severe asthma that required three hospitalizations in the past year. She is now receiving
therapy that has greatly reduced the frequency of severe attacks. Which drug is most likely responsible for this
benefit?
A. Inhaled albuterol
B. Inhaled ipratropium
C. Inhaled fluticasone
D. Oral zafirlukast
Correct answer = C.
Piperacillin/tazobactam (Option B) provides broad-spectrum coverage against both Gram-negative
(Escherichia coli) and anaerobic (Bacteroides fragilis) organisms
3. Which agent is a preferred antihistamine for the management of allergic rhinitis?
A. Chlorpheniramine
B. Diphenhydramine
C. Phenylephrine
D. Cetirizine
Correct answer = D.
Cetirizine (Option D) is a preferred antihistamine for allergic rhinitis due to its effectiveness and lower
sedative effects compared to older antihistamines like chlorpheniramine and diphenhydramine
4. A 58-year-old man with a history of hepatitis C, cirrhosis, and ascites presents with spontaneous bacterial
peritonitis. Which of the following antibiotics requires close monitoring and dosing adjustment in this patient
given his liver disease?
A. Penicillin G.
B. Tobramycin.
C. Erythromycin.
D. Vancomycin.
Correct answer = C.
Erythromycin (Option C) may require close monitoring and dosing adjustment in patients with Liver
disease, particularly those with cirrhosis, due to its potential for hepatic metabolism
5. If 10 mg of oxycodone produces a greater analgesic response than does aspirin at any dose, which is correct?
Correct answer = A.
If oxycodone produces a greater analgesic response than aspirin, it indicates that oxycodone
is more efficacious than aspirin (Option A).
6. In the presence of propranolol, a higher concentration of epinephrine is required to elicit full antiasthmatic
activity. Propranolol has no effect on asthma symptoms. Which is correct regarding these medications?
A. Epinephrine is less efficacious than is propranolol.
B. Epinephrine is a full agonist, and propranolol is a partial agonist.
C. Epinephrine is an agonist, and propranolol is a competitive antagonist.
D. Epinephrine is an agonist, and propranolol is a noncompetitive antagonist.
Correct answer = C.
Epinephrine is an agonist, and propranolol is a competitive antagonist (Option C). Propranolol
blocks the effects of epinephrine, requiring a higher concentration of epinephrine to elicit full antiasthmatic
activity.
7. A β-blocker was prescribed for hypertension in a patient with asthma. After a week of treatment,
the asthma attacks got worse, and the patient was asked to stop taking the β-blocker. Which βblocker would you
suggest as an alternative that is less likely to worsen the asthma?
A. Propranolol
B. Metoprolol
C. Labetalol
D. Carvedilol
Correct answer = B.
Metoprolol (Option B) is a cardioselective B-blocker and is considered safer in patients with
asthma compared to non-selective ß-blockers like propranolol.
Correct answer = B.
Inhibition of calcium absorption (Option B) contributes to osteoporosis with long-term use of
glucocorticoids, leading to decreased bone mineral density.
9. A patient with Addison disease treated with hydrocortisone is experiencing dehydration and
A. Dexamethasone
B. Fludrocortisone
C. Prednisone
D. Triamcinolone
Correct answer = B.
Fludrocortisone (Option B) is a mineralocorticoid that helps retain sodium and water, addressing the
dehydration and hyponatremia commonly seen in Addison's disease.
10. Which contraceptive method provides long-acting reversible contraception (LARC)?
1. An 18-year-old female patient is brought to the emergency department due to drug overdose. Which of
the following routes of administration is the most desirable for administering the antidote for the drug
overdose?
A. Intramuscular
B. Intravenous
C. Oral
D. Subcutaneous
E. Transdermal
Correct answer = B.
Intravenous (IV) administration (Option B) is preferred for the antidote in a drug overdose for rapid and direct
delivery.
2. A 68-year-old woman is brought to the emergency department for treatment of a myocardial infarction.
She is currently taking clopidogrel (antiplatelet agent) and aspirin daily, as well as omeprazole (potent
CYP inhibitor) for heartburn. Which of the following is the most likely contributor to her myocardial
infarction today?
3.. Which of the following best describes how a drug that acts as an agonist at the A subtype of GABA receptors
affects signal transduction in a neuron?
A. Activation of this receptor subtype alters transcription of DNA in the nucleus of the neuron.
B. Activation of this receptor subtype opens ion channels that allow sodium to enter cells and increases the chance
of generating an action potential.
C. Activation of this receptor subtype opens ion channels that allow chloride to enter cells and decreases the
chance of generating an action potential.
D. Activation of this receptor subtype results in G protein activation and increased intracellular second messenger
levels.
Correct answer = C.
Activation of GABA-A receptors opens chloride channels, allowing chloride to enter cells and hyperpolarize the
neuron, decreasing the chance of generating an action potential.
4. In the presence of propranolol, a higher concentration of epinephrine is required to elicit full antiasthmatic
activity. Propranolol has no effect on asthma symptoms. Which is correct regarding these medications?
Correct answer = C.
Epinephrine is an agonist, and propranolol is a competitive antagonist (Option C). Propranolol blocks
the effects of epinephrine, requiring a higher concentration of epinephrine to elicit full antiasthmatic
activity.
5. If there were spare β1-adrenergic receptors on cardiac muscle cells, which statement would be correct?
A. The number of spare β1-adrenergic receptors determines the size of the maximum effect of the agonist
epinephrine.
B. Spare β1 adrenergic receptors make the cardiac tissue less sensitive to epinephrine.
C. A maximal effect of epinephrine is seen when only a portion of β1 adrenergic receptors are occupied.
D. Spare receptors are active even in the absence of epinephrine.
Correct answer = C.
Spare receptors allow a maximal effect even when only a portion of receptors are occupied (Option C).
The presence of spare receptors does not affect sensitivity but influences the magnitude of the response.
6. A 12-year-old girl with asthma presents to the emergency room with complaints of cough, dyspnea, and
wheezing after visiting a riding stable. Which is the most appropriate drug to rapidly reverse her
bronchoconstriction?
A. Inhaled fluticasone
B. Inhaled beclomethasone
C. Inhaled albuterol
D. Intravenous propranolol
Correct answer = C.
Inhaled albuterol (Option C) is a rapid-acting bronchodilator and the most appropriate choice for quickly reversing
bronchoconstriction in asthma.
7. Which part of the adrenal gland is correctly paired with the type of substance it secretes?
A. Adrenal medulla—corticotropin
B. Zona fasciculata—cortisol
C. Zona glomerulosa—androgens D. Zona reticularis—catecholamines
Correct answer = B.
Zona fasciculata secretes cortisol (Option B), not the adrenal medulla, which produces catecholamines.
8. A 75-year-old man with chronic obstructive pulmonary disease is diagnosed with suspected influenza
based on complaints of flu-like symptoms that began 24 hours ago. Which agent is most appropriate to
initiate for the treatment of influenza?
A. Oseltamivir
B. Zanamivir
C. Rimantadine
D. Amantadine
Correct answer = A.
Oseltamivir (Option A) is the most appropriate antiviral agent for influenza treatment.
9. A 62-year-old man with human immunodeficiency virus infection is being treated with an
antiretroviral regimen containing elvitegravir/cobicistat/emtricitabine/tenofovir disoproxil fumarate
and has achieved a sustained undetectable level of HIV RNA. His prescriber would like to change his
therapy to elvitegravir/cobicistat/emtricitabine/tenofovir alafenamide. Which information should the
prescriber provide to the patient that best summarizes the advantage of tenofovir alafenamide over
tenofovir disoproxil fumarate?
Correct answer = D
Tenofovir alafenamide has an improved renal and bone safety profile compared to tenofovir disoproxil fumarate
(Option D).